LSAT and Law School Admissions Forum

Get expert LSAT preparation and law school admissions advice from PowerScore Test Preparation.

 Administrator
PowerScore Staff
  • PowerScore Staff
  • Posts: 8929
  • Joined: Feb 02, 2011
|
#91290
Complete Question Explanation

The correct answer choice is (E).

Answer choice (A):

Answer choice (B):

Answer choice (C):

Answer choice (D):

Answer choice (E) This is the correct answer choice.

This explanation is still in progress. Please post any questions below!
User avatar
 Mr_Churchill
  • Posts: 2
  • Joined: Feb 07, 2022
|
#93685
Do you have an ETA on this explanation? Thank you in advance!
 Adam Tyson
PowerScore Staff
  • PowerScore Staff
  • Posts: 5191
  • Joined: Apr 14, 2011
|
#93687
I'm not sure when we'll have the full diagram and explanation all set up here, Mr_Churchill, but I'll give you a quick rundown using the "Rule by Rule" method of eliminating wrong answers to List questions (the ones that ask which answer is acceptable, meaning the other four answers must all break the rules).

The first rule, K before F, eliminates answer B

The second rule, L before K, doesn't eliminate anything. Don't worry, that's common! Keep moving through the rules.

The third rule, M before J, eliminates C

The fourth rule, if H is 1st then L is 4th, eliminates A

The last rule, if F is 6th then J is 3rd, eliminates D

That leaves E as the only answer that does not break a rule, and that makes it a winner!

Get the most out of your LSAT Prep Plus subscription.

Analyze and track your performance with our Testing and Analytics Package.